1answer.
Ask question
Login Signup
Ask question
All categories
  • English
  • Mathematics
  • Social Studies
  • Business
  • History
  • Health
  • Geography
  • Biology
  • Physics
  • Chemistry
  • Computers and Technology
  • Arts
  • World Languages
  • Spanish
  • French
  • German
  • Advanced Placement (AP)
  • SAT
  • Medicine
  • Law
  • Engineering
vlada-n [284]
3 years ago
13

The graph of the piecewise function f(x) is shown. 1 f(x)

Mathematics
1 answer:
Tema [17]3 years ago
6 0

Answer:

In a word processing document or on a separate piece of paper, use the guide to construct a two-column proof proving that lines l and m are parallel.

You might be interested in
P(A) = 0.8, P(B) = 0.55 and P(A and B) = 0.35.<br> What is P(A or B)?
Likurg_2 [28]
<h3>Answer:  1</h3>

Work Shown:

P(A or B) = P(A) + P(B) - P(A and B)

P(A or B) = 0.8 + 0.55 - 0.35

P(A or B) = 1.35 - 0.35

P(A or B) = 1

A probability of 1 means there is 100% certainty the event happens. This must mean that there are only possible outcomes are: either event A happens, or B happens, or possibly both A and B happen simultaneously.

7 0
2 years ago
Read 2 more answers
Given f(x) = 7x-2, what is the value of f(2)?
motikmotik

Answer:

f(2)=12

Step-by-step explanation:

f(x)=7x-2

Plug 2 into the equation, since it says "f(2)".

f(x)=7(2)-2

f(x)=14-2

f(x)=12, done!

7 0
3 years ago
Read 2 more answers
Consider the numbers 3/10 and 4/13
serg [7]
Make bototm number same
3/10 times 13/13=39/130
4/13 times 10/10=40/130

A.
39/130 and 40/130
they are 1/130 apart


B.
a ratioal number between
39/130=390/1300
40/130=400/1300
between 390/1300 and 400/1300
any number from 391/1300 to 399/1300 example 395/1300


3 0
3 years ago
Hello, Brainly community!
ioda

Answer:

(B)  \displaystyle \frac{W(3.1) - W(2.9)}{0.2}

General Formulas and Concepts:

<u>Calculus</u>

Limits

Derivatives

  • The definition of a derivative is the slope of the tangent line.

Derivative Notation

Instantaneous Rates

  • Tangent Line: \displaystyle f'(x) = \frac{f(b) - f(a)}{b - a}

Step-by-step explanation:

Since we are trying to find a <em>rate</em> at which W(t) changes, we must find the <em>derivative</em> at <em>t</em> = 3.

We are given 2 close answer choices that would have the same <em>numerical</em> answer but different <em>meanings</em>:

  1. (A)  \displaystyle  \lim_{t \to 3} W(t)
  2. (B)  \displaystyle \frac{W(3.1) - W(2.9)}{0.2}

If we look at answer choice (A), we see that our units would simply just be volume. It would not have the units of a rate of change. Yes, it may be the closest numerically correct answer, but it does not tell us the <em>rate</em> at which the volume would be changing and it is not a derivative.

If we look at answer choice (B), we see that our units would be cm³/s, and that is most certainly a rate of change. Answer choice (B) is also a <em>derivative</em> at <em>t</em> = 3, and a derivative tells us what <em>rate</em> something is changing.

∴ Answer choice (B) will give us the best estimate for the value of the instantaneous rate of change of W(t) when <em>t</em> = 3.

Topic: AP Calculus AB/BC (Calculus I/I + II)

Unit: Differentiation

Book: College Calculus 10e

8 0
2 years ago
Giving 15 points for the answer.
max2010maxim [7]

Answer:

4p-3

Explaination

when we subtract

4p-3 -3p-6

it will give us p _9

8 0
2 years ago
Other questions:
  • A bar of metal is cooling from 1000°C to room temperature, 22°C. The temperature, H, of the bar t minutes after it starts coolin
    6·1 answer
  • Lines l, m, and n are parallel.<br><br> What is the value of x?<br><br> Here is the picture.
    14·2 answers
  • Convert 1.05 to a percent.<br> 1.05%<br> 0.105%<br> 10.5%<br> 105%
    12·2 answers
  • How do you solve this for math -9/8+9/8
    10·1 answer
  • a school bus has seating for 75 students. There are 13 buses at a school. what is the maximum number of students the buses can s
    7·2 answers
  • How to use the distrubtive property to multiply a number by a sum
    12·1 answer
  • Can someone please explain to me how to solve #7?
    6·1 answer
  • Can someone please help me !!!
    11·1 answer
  • I NEED HELP ASAP!
    7·1 answer
  • M/5=3(2m+1) verbal expression
    11·1 answer
Add answer
Login
Not registered? Fast signup
Signup
Login Signup
Ask question!